pls help me in this question it is really needed

Answers

Answer 1

Answer:

6 1/8 ×10 2/7= 60

60÷2 1/3= 30

The answer:

30


Related Questions

Please helpppppp I need to pass

Answers

Answer:

x = -1.4 and x=2

Step-by-step explanation:

The solutions are where the graphs intersect

The graphs appear to intersect at x = -1.4 and x=2

translate into a variable expression and then simplify. five times the sum of a number and four​

Answers

Answer:

5(n+4)

5n+20

Step-by-step explanation:

Let n be the number

5* (n+4)

Distribute

5n+20

Please help! Thank you!
In the figure below, O is the center of the circle. Name a tangent of the circle.
A. AO
B. FG
C. AB
D. HK

Answers

Answer:

Its A

Step-by-step explanation:

The tangent of the given circle is FG hence the correct option will be an option (B).

What is a tangent of a circle?

The tangent of a circle is a line that intersects the circle at the periphery of the circle.

If you draw a line that goes through the center to the tangent touching point then it will give you a 90-degree angle.

Given the circle it's clear that the tangent is only FG hence it will be the correct option  

A circle can have an infinite number of tangents.

In other words, a straight line that only touches a circle twice is said to be tangent to it. The term point of intersection refers to this location

At the tangent line, the tangent to a circle is orthogonal to the radius.

For more information about the tangent of the circle

https://brainly.com/question/23265136

#SPJ2

complete the table below?

Answers

ABC = $425

Load = 0

Total Cost = $425

Sales Price = $850

Sales price ÷ total cost = 850/425

= 2%

DEF = $600

Load = $375

Total Cost = $975

Sales Price = $1200

Sales Price ÷ Total cost = 1200/975

= 1% (Nearest 1%)

Must click thanks and mark brainliest

Which of the following represents 32/100? A. thirty-two hundredths B. 0.032 C. 0.23 D. thrity-two tenths​

Answers

Answer:

A

Step-by-step explanation:

32 hundredths is 32/100

Side note

32/100 can be simplified to 8/25.

A factory used 99.19 kilograms of tomatoes to make 7 batches of pasta sauce. What quantity of tomatoes did the factory put in each batch?

Answers

Answer:

14.17 kilograms

Step-by-step explanation:

Find what quantity of tomatoes was in each batch by dividing the total amount of tomatoes by the number of batches:

99.19/7

= 14.17

So, the factory put 14.17 kilograms of tomatoes in each batch.

Find the value of x in each case:

Answers

<FIG=2x(Opposite angles)<FGI=x(Interior angles)

We know

Sum of two interior angles =exterior angle

[tex]\\ \sf\longmapsto 2x+x=3x[/tex]

[tex]\\ \sf\longmapsto 3x=3x[/tex]

[tex]\\ \sf\longmapsto x=1[/tex]

The 3rd and 6th term of a geometric progression are 9/2 and 243/16 respectively find the first term, common ratio, seventh term​

Answers

Answer:

Hello,

Step-by-step explanation:

[tex]Let\ (u_n)\ the\ geometric\ progression.\\\\r\ is\ the\ common\ ratio.\\\\u_3=u_0*r^3\\u_6=u_0*r^6\\\\\dfrac{u_6}{u_3} =r^3=\dfrac{\frac{243}{16} }{\frac{9}{2} } =\dfrac{27}{8} =(\frac{3}{2} )^3\\\\\boxed{r=\dfrac{3}{2} }\\\\\\u_3=u_1*r^2 \Longrightarrow\ u_1=\dfrac{u_3}{r^2} =\dfrac{\frac{9}{2} }{(\frac{3}{2^2}) } =2\\\\\\u_7=u_6*\dfrac{3}{2} =\dfrac{729}{32}[/tex]

Which graph represents the function f(x) = x-2?

Answers

Answer:

click in photo

nejdjjd

nxndjdbbdjf

lấy x=0 ta có y= -2 => A(0;-2)

lấy y=0 ta có x=2 => B(2;0)

nối 2 điểm A và B ta có đồ thị:

1. Find the 4th term for the sequence with formula tn= n² + 1

Answers

Answer:

17

Step-by-step explanation:

T4 = 4² + 1

T4 = 4² + 1 = 17

Yip yip that's all

90units needed 8 units per case what's the #of cases & # of additional units

Answers

Answer:

# of cases: 11

Additional units: 2

Step-by-step explanation:

If each case can hold 8 units, and we want to find the total # number of cases, we have to divide the # of units (8) for one case by the total # units (90).

As you can see, after dividing by 8, we have a total of 11 cases and a remainder of 2 units. The remainder will be the # of additional units because we cannot have another case filled with 8 units.

The object of a popular carnival game is to roll a ball up an incline into regions with different
values. The probability that Angus will get 100 points in a roll is 40%, 200 points is 35%, and
300 points is 25%. Find the expected value, E(X), of a roll.
O 185
O 200
O 400
O 150

Answers

The expected value, E(x) of the given observation is 185

The expected value is the mean of the overall observed value or random value. In other words, it is the average of the observed values.

The given parameters can be represented as:

[tex]\begin{array}{cccc}x & {100} & {200} & {300} \ \\ P(x) & {40\%} & {35\%} & {25\%} \ \end{array}[/tex]

The following formula calculates the expected value:

[tex]E(x) =\sum x * P(x)[/tex]

So, we have:

[tex]E(x) = 100 * 40\% + 200 * 35\% + 300 * 25\%[/tex]

[tex]E(x) = 40 + 70 + 75[/tex]

[tex]E(x) = 185[/tex]

Learn more about expected values at:

https://brainly.com/question/16726343

anyone know the answers for the final exam for part one of algebra 2 on edg?

Answers

Answer:

just show the questions i will help

Step-by-step explanation:

First make a substitution and then use integration by parts to evaluate the integral. integral t^11 e^-t^6 dt + C

Answers

It looks like you want to find

[tex]\displaystyle \int t^{11} e^{-t^6}\,\mathrm dt[/tex]

Substitute u = -t ⁶ and du = -6t ⁵ dt. Then

[tex]\displaystyle \int t^{11} e^{-t^6}\,\mathrm dt = \frac16 \int (-6t^5) \times (-t^6) e^{-t^6}\,\mathrm dt = \frac16 \int ue^u \,\mathrm du[/tex]

Integrate by parts, taking

f = u   ==>   df = du

dg = eᵘ du   ==>   g = eᵘ

Then

[tex]\displaystyle \frac16 \int ue^u \,\mathrm du = \frac16\left(fg-\int g\,\mathrm df\right) \\\\ =\frac16 ue^u - \frac16\int e^u\,\mathrm du \\\\ =\frac16 ue^u - \frac16 e^u + C \\\\ =-\frac16 t^6 e^{-t^6} - \frac16 e^{-t^6} + C \\\\ =\boxed{-\frac16 e^{-t^6} \left(t^6+1\right) + C}[/tex]

Start with the number 2380.
Divide by 10,
The 8 will end up in the _____ place.

Answers

Answer:

The 8 will end up in the ones place.

Step-by-step explanation:

If you want to divide 2380 by 10… sry gtg

The 8 will end up in the "ones place".

How can we interpret the division?

When 'a' is divided by 'b', then the result we get from the division is the part of 'a' that each one of 'b' items will get. Division can be interpreted as equally dividing the number that is being divided into total x parts, where x is the number of parts the given number is divided.

We need to find the 8 will end up in which place

A negative divided by a negative is positive, then;

2380/ 10 = 238

Therefore, The 8 will end up in the _ ones_ place.

Learn more about division here:

brainly.com/question/26411682

#SPJ2

i need help on 8-9 plss :))

Answers

Answer:

8. SU = 24

9. TU = 16√3

Step-by-step explanation:

Recall: SOH CAH TOA

8. Reference angle (θ) = 30°

Opposite = 8√3

Adjacent = SU

Apply TOA,

Tan θ = Opp/Adj

Substitute

Tan 30° = 8√3/SU

Tan 30° × SU = 8√3

SU = 8√3/Tan 30°

SU = 8√3/(1/√3) (tan 30° = 1/√3)

SU = 8√3*√3/1

SU = 8*3

SU = 24

9. Reference angle (θ) = 30°

Opposite = 8√3

Hypotenuse = TU

Apply SOH,

Sin θ = Opp/Hyp

Substitute

Sin 30° = 8√3/TU

Sin 30° × TU = 8√3

TU = 8√3/sin 30°

TU = 8√3/(½) (sin 30° = ½)

TU = 8√3 × 2/1

TU = 16√3

Please help im new and i need help!
Please help me if you onlw the answers please!!

Answers

9514 1404 393

Answer:

  a) 2.038 seconds

  b) 5.918 meters

  c) 1.076 seconds

Step-by-step explanation:

For the purpose of answering these questions, it is convenient to put the given equation into vertex form.

  h = -4.9t² +9.2t +1.6

  = -4.9(t² -(9.2/4.9)t) +1.6

  = -4.9(t² -(9.2/4.9)t +(4.6/4.9)²) +1.6 +4.9(4.6/4.9)²

  = -4.9(t -46/49)² +290/49

__

a) To find h = 0, we solve ...

  0 = -4.9(t -46/49)² +290/49

  290/240.1 = (t -46/49)² . . . . subtract 290/49 and divide by -4.9

  √(2900/2401) +46/49 = t ≈ 2.0378 . . . . seconds

The ball takes about 2.038 seconds to fall to the ground.

__

b) The maximum height is the h value at the vertex of the function. It is the value of h when the squared term is zero:

  290/49 m ≈ 5.918 m

The maximum height of the ball is about 5.918 m.

__

c) We want to find t for h ≥ 4.5.

  h ≥ 4.5

  -4.9(t -46/49)² +290/49 ≥ 4.5

Subtracting 290/49 and dividing by -4.9, we have ...

  (t -46/49)² ≥ 695/2401

Taking the square root, and adding 46/49, we find the time interval to be ...

  -√(695/2401) +46/49 ≤ t ≤ √(695/2401) +46/49

The difference between the interval end points is the time above 4.5 meters. That difference is ...

  2√(695/2401) ≈ 1.076 . . . . seconds

The ball is at or above 4.5 meters for about 1.076 seconds.

__

I like a graphing calculator for its ability to answer these questions quickly and easily. The essentials for answering this question involve typing a couple of equations and highlighting a few points on the graph.

_____

Additional comment

I have a preference for "exact" answers where possible, so have used fractions, rather than their rounded decimal equivalents. The calculator I use deals with these fairly nicely. Unfortunately, the mess of numbers can tend to obscure the working.

"Vertex form" for a quadratic is ...

  y = a(x -h)² +k . . . .  where the vertex is (h, k) and 'a' is a vertical scale factor.

In the above, we have 'a' = -4.9, and (h, k) = (46/49, 290/49) ≈ (0.939, 5.918)

Write an expression for the baseball team’s Purchase.

Answers

Sorry I do not get what you are asking

Alejandro wants to adopt a puppy from an animal shelter. At the​ shelter, he finds eight puppies that he​ likes: a male and female puppy from each of the four breeds of and Labrador. The puppies are each so cute that Alejandro cannot make up his​ mind, so he decides to pick the dog randomly. Find the probability that Alejandro chooses a .

Answers

Answer:

Hence the required probability is, 3/4

Step-by-step explanation:  

At the shelter, he likes :  

a male coolie, a female coolie, a male boxer, a female boxer, a male beagle, a female beagle, a male Labrador, and a female Labrador.  

Let, A denote the event of selecting a male coolie and B denote the event of selecting a male Labrador.  

P(A) = 1/8 = P(B)  

Here the probability of selecting a puppy except A & B is,  

P(AUB)c = 1 - P(AUB) = 1 - { P(A) + P(B) } = 1 - 1/8 - 1/8 = 3/4

Help!!! QUICK! What is the pattern of the exponents on the a terms in Pascal's Triangle?


A. The largest exponent value of the a terms is equal to one more than the value of the exponent on the binomial. The exponent values then decrease from left to right.


B. The largest exponent value of the a terms is equal to the value of the exponent on the binomial. The exponent values then decrease from left to right.


C. The largest exponent value of the a terms is equal to the value of the exponent on the binomial. The exponent values are then equal to 0 throughout the expansion.


D.The largest exponent value of the a terms is equal to one more than the value of the exponent on the binomial. The exponent values are then equal to 1 throughout the expansion.

Answers

Answer:

B. The largest exponent value of the a terms is equal to the value of the exponent on the binomial. The exponent values then decrease from left to right.

Step-by-step explanation:

The exponent values of the a terms increase from one side of the binomial to the other. The value of the largest exponent is equal to part of the binomial expression.

Quick!!
Shandra has a sheet of cardboard whose area is x2 + 2x - 15 square inches. We know that the length of the cardboard
sheet is (x + 5) inches. What is its width? (Hint: area = length width)

Answers

Answer:

(x - 3) inches

Step-by-step explanation:

the ratio of -15 (of the area term) and +5 (of the length term) suggests a term for the width ending in -3. and we need an x in there to get x² for the area.

so, immediate assumption is (x-3) for the width.

let's check

(x+5)(x-3) = x² + 5x - 3x - 15 = x² + 2x - 15

correct.

f(x)=1/3x+7 find inverse

Answers

Answer:

Step-by-step explanation:

A right pyramid with a square base has
volume of 252 cubic centimeters. The
length of one of the sides of its base is 6
centimeters. Rounded to the nearest
centimeter, what is the vertical height of
the pyramid?

Answers

Hey there!

A right pyramid with a square base just means that it isn't slanted all funny. If you create a triangle with a point on one of the edges of the base, the center of the base, and the top of the pyramid, it would be a right triangle.

To find the volume of a right pyramid, you just take the base area, multiply it by the height, and then divide by three.

However, we are looking for the height. We have been given, so we will just go backwards.

252×3=756 (multiply instead of divide)

6×6=36 (base is a square, so you just square 6. This is our base area)

756/36=21

So, the vertical height of the pyramid is 21 cubic centimeters.

Have a wonderful day! :D

Helpp me plzzz im being timed

Answers

Above question
Slope = change in y / change in x
(2 different random point on the line)
Let’s point out 2 points: (0,3) and (6,0)
So slope = (3 - 0) / (0 - 6) = 3/(-6) = -1/2

Y-intercept is when x = 0, the line cross the y-axis at y = ?
So y = 3
Below question is farmiliar:
Slope = (30-50) / (0-6) = -20/(-6) = 10/3
Y-intercept = 30

17 x 35=56 over 76 hope it helps s

How many solutions exist for the given equation?
121 x+ 1 =3(4x+1)-2
zero
one
two
infinitely many

Answers

Answer: infinitely many

Which function describes this graph

Answers

Answer:

C.

Step-by-step explanation:

A P E X

URGENT PLEASE HELP DUE TMMR MORNING Half of the difference between a number and 3 is 5. What is the number?

Answers

Answer:

Not sure but I think it's 3.75

If average of the numbers 3,9,5,7 and Q is 5 times the value of Q, find the value of Q

Answers

Answer:

q=6

Step-by-step explanation:

3+9+5+7+Q=5Q

5Q-Q=24

Q=6

180 °
35 °
X °

X = ? °

Answers

Answer:

x=145°

Step-by-step explanation:

180 = 35 + x

x = 180-35

x= 145°

find the greatest number that divides 56 and 84 exactly​

Answers

Answer:

28

Step-by-step explanation:

Find the gcf

Other Questions
the mean salary if of 5 employees is $35900. the median is $37000. the mode is $382000. If the median payed employee gets a $3100 raise, thenNew median:New mode: Please help meeeeFind the value of d.A. 47B. 56C. 75D. 30 Figgins is the dean of a college. He appointed Sue to be acting dean while he was out of the country and posted an announcement on the college website which said that Sue was authorized to act in his place. Figgins also told Sue privately that she could do most everything in his job description while he was away, but she did not have the right to make admissions decisions. While Figgins was gone, Sue overruled the admissions committee to admit the child of a wealthy alumnus. Does the child have the right to attend this college 1.- Estaban demasiado cansados para continuara) expresob) tcitonominal Admiral David Farragut led a ___ of about 40 ships past the stronghold forts of Jackson and St. Philip at the mouth of the Mississippi River to transport troops and supplies to capture the cities along the river.A. FleetB. Garrison C. ConvoyD. Caravan Secant ZV intersects secant zw at point Z. Find the length of Tw. If necessary, round to the hundredths place.A. 7B. 8C. 9D. 10 Which of the following is NOT an effective way to manage stress? Focus on goals Reduce sleep time Stay organized Talk to a friend 6th grade Geometry, please help it's due soon! 2. How does a good society get its recognition? You make a purchase at a local hardware store, but what you've bought is too big to take home in your car. For a small fee, you arrange to have the hardware store deliver your purchase for you. You pay for your purchase, plus the sales taxes, plus the fee. The taxes are 7.5% and the fee is $20. (i) Write a function t(x) for the total, after taxes, on the purchase amount x. Write another function f(x) for the total, including the delivery fee, on the purchase amount x. (ii) Calculate and interpret (f o t)(x) and (t o f )(x). Which results in a lower cost to you Elderly woman purchases a table from a local furniture store for $5,000. She decides she does not want the table because of a defect. The woman successfully cancels the payment on the credit card for payment of the table, and continues to keep the table despite the store requesting to pick the table up from woman. What statement is true: Which of the following is NOT true about Charlemagne?A. He was the first Holy Roman Emperor.B. He wanted to reunite the former Roman Empire and linked much of the former territory.C. He was the head of the Merovingian Dynasty.D. He created a bureaucracy with standardized weights and measures and acted as an imperial ruler.E. He led over 50 successful military campaigns, including defeating the Lombard s, Franks, and Saxons, which allowedhim to spread Christianity. Write and solve a word problem that can be modeled by addition of two negative integers. Henry was in a serious accident. Initially after the accident he was very distressed about his loss of friends but later his sorrow lessened. From an interdependency perspective, his initial distress followed by his increasingly positive feelings can best be explained by:___. Kayla parks her car at the corner of Ogilvie and Montreal Rd. She walks 80m East and then turns 30 to the left towards her office building and continues walking for another 100m until she reaches her building. She then takes the elevator to her office 60m above ground level and looks out the window. She can see her car from here. How far is it from where she is to her car in a direct line? solue for &X(3 + X) = 3x + x PLEASE PLEASE HELP PLEASE PLEASE HELP ME WITH THIS QUESTION PLEASE HELP PLEASE HELP PLEASE!!!!The median age for a first marriage in the United States for women was 25.9 in 2009 and 26.1 in 2010. Use an exponential model to predict the median age for women in 2019, where x is the number of years since 2009.A) 23.9B) 28.3C) 28.0D) 24.0 In the reaction HCI + NH4OH --> NH4CI+H2O, which compound has an element ratio of 1:4:1?H2ONH4ClHCI4 Menciona 3 ideas para evitar la contaminacion y preservar el medio ambiente